Thread Rating:
  • 0 Vote(s) - 0 Average
  • 1
  • 2
  • 3
  • 4
  • 5
nbme 1 - rizowana
#31
27. An asymptomatic 21-year-old woman is found to have an adnexal mass on pelvic examination. She uses oral contraceptives. A photograph of the mass is shown. Which of the following is the most likely diagnosis?

A
) Benign cystic teratoma

B
) Corpus luteum cyst

C
) Dysgerminoma

D
) Endometrioma

E
) Mucinous cystoadenoma
Reply
#32
might be AA.?........i have no idea about photo
Reply
#33
30. One month after undergoing an uneventful renal transplant for chronic renal failure secondary to glomerulonephritis, a 38-year-old woman is hospitalized because of increased serum urea nitrogen (BUN) and creatinine levels. Prior to transplantation, she had been receiving hemodialysis for 3 years. Current medications include cyclosporine and prednisone. Examination shows no abnormalities. Over the past 48 hours, urine output has remained stable. Both renal biopsy and a radionuclide scan confirm the diagnosis of acute rejection. Which of the following is the most effective treatment?

A
) Immediate discontinuation of cyclosporine

B
) Increased dosage of corticosteroids

C
) Diuresis and alkalinization of the urine

D
) Renal dialysis for 1–2 weeks

E
) Transplant nephrectomy
Reply
#34
its B right?
Reply
#35
AA.................SE of cyclosporine
Reply
#36
rizowana, I think it is B, like you

this is a case of acute rejection, we have to increase steroid dose
Reply
#37
@DALIA2010...WHAT ABOUT q27.....please
Reply
#38
in Q 27 it can not be B as pt is on pill, so no ovulation, no corpus luteum, not sure about other choices
Reply
#39
31. An 18-year-old man comes to the physician 1 week after he had a blood pressure of 140/110 mm Hg during a routine precollege examination. His temperature is 37.1 C (98.7 F), blood pressure is 140/100 mm Hg, pulse is 92/min, and respirations are 12/min. The upper extremities appear to be more muscular than the lower extremities. Radial pulses are normal; femoral, posterior tibial, and dorsalis pedis pulses are decreased. A grade 2/6 systolic murmur is heard over the precordium, anterior chest, and back. An ECG shows left ventricular hypertrophy. Which of the following is the most appropriate next step in management?

A ) Limiting physical activity

B ) Repeat blood pressure measurement in 1 month

C ) Initiate a low-sodium diet and exercise program

D ) Pharmacologic management

E ) Operative treatment
Reply
#40
EE.......COARCTATION OF AORTA....ADULT TYPE
Reply
« Next Oldest | Next Newest »


Forum Jump: